9529e5d3fb9065f98b113dc3454f11df48be144d
[course.git] / latex / problems / Serway_and_Jewett_4 / problem14.08.tex
1 \begin{problem*}{14.8}
2 Two loudspeakers are placed on a wall $2.00\U{m}$ apart.  A listener
3 stands $3.00\U{m}$ from the wall directly in front of one of the
4 speakers.  A single oscillator is driving the speakers at a frequency
5 of $300\U{Hz}$.  \Part{a} What is the phase difference between the two
6 waves when they reach the observer?  \Part{b} What is the frequency
7 closest to $300\U{Hz}$ to which the oscillator may be adjusted so that
8 the observer hears minimal sound?
9 \end{problem*} % problem 14.8
10
11 \begin{solution}
12 \begin{center}
13 \begin{asy}
14 import Mechanics;
15
16 real u=1cm;
17 pair SpeakerA = (0,0);
18 pair SpeakerB = (0,2u);
19 pair Listener = (3u,0);
20
21 // Extra label text for spacing with inline asymptote
22 Distance dAB = Distance(SpeakerA, SpeakerB, Label("$L=2\U{m}$", "L=2 m"));
23 Distance dAL = Distance(Listener, SpeakerA, Label("$d_a=3\U{m}$", "da=3 m"));
24 Distance dBL = Distance(SpeakerB,Listener,
25                         Label("$\qquad\qquad\qquad\qquad\qquad d_b=\sqrt{L^2+d_a^2}=3.606\U{m}$",
26                               "$d_b=\sqrt{L^2+d_a^2}=3.606mm$"));
27
28 dot(SpeakerA);
29 label("$S_a$", SpeakerA, W);
30 dot(SpeakerB);
31 label("$S_b$", SpeakerB, W);
32 dot(Listener);
33 label("Listener", Listener, E);
34 dAB.draw();
35 dAL.draw(rotateLabel=false);
36 dBL.draw(rotateLabel=false);
37 \end{asy}
38 \end{center}
39 \Part{a}
40 From Table 13.1 we find that the speed of sound in air at $20\dg C$ is
41 $v=343\U{m/s}$.  The wavelength of this sound is
42 \begin{equation}
43   \lambda = \frac{v}{f} = 1.143\U{m} 
44 \end{equation}
45 The phase change from speaker $S_a$ is therefore
46 \begin{equation}
47   \theta_a = k d_a = \frac{2\pi d_a}{\lambda} = 16.49\U{rad} \;,
48 \end{equation}
49 and from speaker $S_b$ is
50 \begin{equation}
51   \theta_b = k d_b = \frac{2\pi d_b}{\lambda} = 19.81\U{rad} \;.
52 \end{equation}
53 The phase difference is
54 \begin{equation}
55   \Delta\theta = \theta_b - \theta_a = \ans{3.33\U{rad}}
56 \end{equation}
57
58 \Part{b}
59 For minimal sound, we want the phase difference to be exactly $\pi$
60 (or some odd multiple of $\pi$).  We see that it's already close to
61 $\pi$ with our initial frequency of $300\U{Hz}$, only a bit high.
62 Decreasing the freqency a bit will reduce the rate of dephasing
63 between the two waves, reducing $\Delta\theta$, so we're looking for a
64 frequency slightly less than $300\U{Hz}$.
65 \begin{align}
66   \Delta\theta &= \theta_b-\theta_a
67     = \frac{2\pi (d_b-d_a)}{\lambda}
68     = \frac{2\pi f (d_b-d_a)}{v} \\
69   f &= \frac{v\Delta\theta}{2\pi (d_b-d_a)}
70     = \frac{v \pi}{2\pi (d_b-d_a)}
71     = \frac{v}{2(d_b-d_a)}
72     = \frac{343\U{m/s}}{2\cdot 0.606\U{m}} = \ans{283\U{Hz}} \;.
73 \end{align}
74 \end{solution}